Stetigkeit - Maximum Minimum

Neue Frage »

Yoko Auf diesen Beitrag antworten »
Stetigkeit - Maximum Minimum
Hallo,

ich habe immer Probleme mit Aufgaben wo keine wirkliche Funktion gegeben ist.
Also auch mit der folgenden.

Sei stetig und es gelte



Dann besitzt f ein Minimum oder ein Maximum und |f| besitzt ein Maximum auf IR.

Mir ist die Idee mit dem Satz von Weierstraß gekommen. Weil der besagt ja, laut dem Teubner Taschenbuch, das jede stetige Funktion ein Minimum und ein Maximum besitzt.
Also das es Punkte gibt.

Dann würde ich folgendes erhalten.
Aber hier weiß ich nicht weiter, da das Intervall ja von unendlich bis -undendlich läuft.
Ansonsten fällt mir da noch die Differentialrechnung ein, aber es mir nicht weiter.

Vielleicht würde eine Erklärung der Aufgabe zu meinem Verständnis schon etwas weiter helfen.

Gruß Yoko
Poff Auf diesen Beitrag antworten »
RE: Stetigkeit - Maximum Minimum
Dann besitzt f ein Minimum oder ein Maximum und |f| besitzt ein Maximum auf IR.

das ist falsch so wie das da steht (modulo Definition von Min und Max)

nimm f = 0
Leopold Auf diesen Beitrag antworten »

Da man bei Minimum/Maximum auch immer die Gleichheit zuläßt, ist das kein Gegenbeispiel. Der Satz ist richtig.
Poff Auf diesen Beitrag antworten »

und |f| besitzt ein Maximum auf IR.

schließt natürlich nicht aus das |f| auch ein Minimum besitzt

ich weiß ich kenn den Krampf
Yoko Auf diesen Beitrag antworten »
RE: Stetigkeit - Maximum Minimum
das ist falsch so wie das da steht (modulo Definition von Min und Max)

Soll das heißen das die Aufgabe falsch gestellt ist? Wie lautet denn die Modulo Def von Min und Max?

gruß yoko
Poff Auf diesen Beitrag antworten »
RE: Stetigkeit - Maximum Minimum
Nein, lies Leopolds Beitrag
 
 
Yoko Auf diesen Beitrag antworten »

angenommen ich wolle zeigen das das mit dem minimun und maximum stimmt wenn die lim sache gilt
wie gehe ich dann mit dem f(x) um?
Poff Auf diesen Beitrag antworten »

Die Sache sagt vereinfacht folgendes aus:
f kann im endlichen Bereich nicht unendlich werden weil f stetig ist
und im unendlichen Bereich kann f nicht unendlich werden wegen
der vorgegebenen Grenzwerte. Demnach muss f nach oben oder
unten beschränkt sein, aber das wirst ja schon gewusst haben . Augenzwinkern
.

für alle x0 aus R existiert f(x0), weil wegen der Stetigkeit
f(x0)= lim[x->x0] f(x) gilt muss f(x0) existieren und zwar endlich.

Wegen lim[x->+-oo]f = 0 gibt es zu jedem d>0 xd aus R sodass
gilt |f(xd)|< d für alle x links -xd und recht +xd

auf deutsch f wird rechts und links dieser Schranken nicht mehr
'größer' als d

wie man nun argumentieren kann dass sie ihren Extremwert als HP
auch miteinschließen muss fällt mir derweil nicht ein, jedenfalls ist
sie auf endlichem Bereich endlich und hat daher ein Sup oder Inf usw.


will mich garnicht in dieses Glump begeben ...
ich denke wenn du 'deinen Satz' bemühst kommst leichter über
die Runden weil du dir über den anderen Quatsch keine Gedanken
machen brauchst, so in dem Stil ... Augenzwinkern
.


ich finde es ekelhaft wenn man die Mathematik zu ernst nimmt
(das ist gegen keine Person gerichtet oder gedacht und schon
garnicht gegen dich, sondern ist einer meiner Standpunkte)

---------------------------------------------------------------








Mich leicht in Schuld fühlend will ich nochmal anders drauf antworten.
Mal unterstellt dein Satz von oben leistet das was du vorgibst,
dass er es leiste, dann kannst das so machen

Nimm ein beliebiges abgeschlossenes Intervall [a,b] mit f<>0 dann
hat f dort ein Max. u. Minimum nach deinem Satz. Der 'Einfachheit'
halber will ich mich nur mal um das Max M kümmern, notfalls sei es
eben für ein (-f) zu betrachten.
Nun gibt es wegen der Existenz der beiden Grenzwerte Zahlen xm
aus R sodass gilt

Für alle x < -xm und x > +xm ist |f(x)| < M

das heißt aber nichts anderes als dass |f| außerhalb des Intervall
[-xm,xm] kleiner M ist und mindestens M innerhalb des Intervalles
erreicht wird sofern nur a,b auch innerhalb liegen. Liegt a oder b
außerhalb dann wird das Intervall [-xm,xm] einfach soweit nach
rechts und links erweitert dass sie nun innerhalb von [-xm',xm'] liegen.

Nach deinem Satz muss die Fkt f nun in diesem Intervall ein
Max und Min haben und wegen der Konstuktion ist dies auch
das globale Maximum.


sieh wie du's verbessern und was du daraus machen kannst, ich
hoffe es ist wenigstens verständlich
Yoko Auf diesen Beitrag antworten »

hi,

könntest du mir sagen wie die Funktion aussehen könnte wo der Grenzwert beidseitig gegen null läuft?

Irgendwie fehlt mir das Verständnis zu dieser Aufgabe obwohl die recht simple scheint.

Gruß Yoko
AD Auf diesen Beitrag antworten »

Ein passendes Beispiel wäre
Leopold Auf diesen Beitrag antworten »

Oder noch einfacher:


Neue Frage »
Antworten »



Verwandte Themen

Die Beliebtesten »
Die Größten »
Die Neuesten »